Tài liệu hạn chế xem trước, để xem đầy đủ mời bạn chọn Tải xuống
1
/ 23 trang
THÔNG TIN TÀI LIỆU
Thông tin cơ bản
Định dạng
Số trang
23
Dung lượng
285,49 KB
Nội dung
MỘT SỐ VẤN ĐỀ VỀ BẤT ĐẲNG THỨC BẬC BỐN BA BIẾN Nguyễn Văn Huyện1 Tống Hữu Nhân2 1. Mở đầu Trên tạp chí IJPAM định lý: nhóm tác giả Vasile Cirtoaje Võ Quốc Bá Cẩn có đề xuất Với a, b, c ba số thực, xét đa thức F (a, b, c) = a4 + A b2 c2 + Babc a+C Nếu + A + B + C + D = 0, bất đẳng thức F (a, b, c) 3(1 + A) b3 c + D bc3 . C + CD + D2 . Với định lý lớp toán bậc ba biến số giải triệt để. Tuy nhiên việc chứng minh áp dụng kỳ thi điều dễ dàng. Theo quan điểm cá nhân định lý giống tiêu chuẩn việc sáng tạo bất đẳng thức phương pháp để giải toán. Điều mà quan tâm không định lý tổng quát mà công cụ không quá phức Sinh viên trường Đại học Giao Thông Vận Tải thành phố Hồ Chí Minh. Sinh viên trường Đại học Y Dược thành phố Hồ Chí Minh. International Journal of Pure and Applied Mathematics Trường đại học Department of Automatic Control and Computers, Ploiesti, Romania Hà Nội Một số vấn đề bất đẳng thức bậc bốn ba biến tạp để sử dụng phòng thi. Vì thế, viết nhỏ này, xin chia sẻ vài kỹ thuật nhỏ hiệu việc xử lý bất đẳng thức bậc ba biến. 2. Các toán đối xứng Năm 2011 diễn đàn toán học artofproblemsolving.com thành viên mudok có đề xuất toán sau đây: Bài 2.1. Cho số thực a, b, c thỏa mãn a + b + c = 0. Chứng minh (ab + bc + ca)2 + 9abc 3(ab + bc + ca). (2.1.1) Lời giải. Cách tự nhiên ta rút c = −a − b thay vào (2.1.1), ab − (a + b)2 ab − (a + b)2 , − 9ab(a + b) (a2 + ab + b2 )2 + 3(a2 + ab + b2 ) 9ab(a + b). Quan sát chút ta thấy bất đẳng thức lúc chứa ba đại lượng a2 + ab + b2 , ab, a + b chúng có mối liên hệ với thông qua đánh giá a2 + ab + b2 (a + b)2 3ab, suy (a2 + ab + b2 )2 + 3(a2 + ab + b2 ) (a + b)4 + 9ab. 16 Như ta cần chứng minh (a + b)4 + ab 16 9ab(a + b). Rất tiếc bất đẳng thức không (có thể kiểm tra với a = −1, b = 1). Do ta đổi lại đánh giá theo kiểu (a2 + ab + b2 )2 + 3(a2 + ab + b2 ) (3ab)2 + · (a + b)2 = 9a2 b2 + (a + b)2 , (2.1.2) quy toán chứng minh a2 b + (a + b)2 ab(a + b). Bất đẳng thức theo bất đẳng thức AM-GM, có lỗi đánh giá (2.1.2) (a2 + ab + b2 )2 (3ab)2 ab 0, ta khắc phục lỗi kỹ thuật sau. Do ab · bc · ca = a2 b2 c2 nên ba số ab, bc, ca có số không âm, giả sử ab 0. Với lập luận trên, áp dụng bất đẳng thức AM-GM, ta có a2 + ab + b2 (a + b)2 3ab 0. (2.1.3) Một số vấn đề bất đẳng thức bậc bốn ba biến Suy (a2 + ab + b2 )2 + 3(a2 + ab + b2 ) 9a2 b2 + 9(a + b)2 . Ta chứng minh a2 b + (a + b)2 ab(a + b). Cũng theo bất đẳng thức AM-GM (a + b)2 ab + 2 a2 b · (a + b)2 = ab |a + b| ab(a + b). (2.1.4) Đẳng thức xảy (2.1.3) (2.1.4) trở thành đẳng thức, tức a = b a2 b2 = (a + b) Giải hệ ta a = b = c = a = b = 1, c = −2. Bài chứng minh. Bài 2.2. Cho ba số thực a, b, c thỏa mãn a + b + c = 3. Chứng minh 3(a4 + b4 + c4 ) + a2 + b2 + c2 + 6(a3 + b3 + c3 ). (2.2.1) Lời giải. Đây toán đối xứng, nên ta tìm cách đưa điều kiện dạng tổng 0, dùng phép trên. Rất tự nhiên đặt a = x + 1, b = y + 1, c = z + 1, x + y + z = (a − 1) + (b − 1) + (c − 1) = 0. Với phép đặt a2 + b2 + c2 = (x + 1)2 + (y + 1)2 + (z + 1)2 = x2 + y + z + 2(x + y + z) + = x2 + y + z + 3, tương tự a3 + b3 + c3 = x3 + y + z + 3(x2 + y + z ) + 3, a4 + b4 + c4 = x4 + y + z + 4(x3 + y + z ) + 6(x2 + y + z ) + 3. Bất đẳng thức (2.2.1) trở thành 3(x4 + y + z ) + 6(x3 + y + z ) + x2 + y + z Giả sử xy 0. (2.2.2) 0, thay z = −x − y vào (2.2.2), ta 3[x4 + y + (x + y)4 ] + 6[x3 + y − (x + y)3 ] + x2 + y + (x + y)2 3(x4 + 2x3 y + 3x2 y + 2xy + y ) + x2 + xy + y 3(x2 + xy + y )2 + x2 + xy + y 0, 9xy(x + y) 9xy(x + y). Một số vấn đề bất đẳng thức bậc bốn ba biến Áp dụng bất đẳng thức AM-GM, ta có x2 + xy + y (x + y)2 3xy 0, (2.2.3) suy 3(x2 + xy + y )2 + x2 + xy + y 27x2 y + (x + y)2 . Do ta cần chứng minh 9x2 y + (x + y)2 3xy(x + y). Cũng theo bất đẳng thức AM-GM, 9x2 y + (x + y)2 9x2 y · (x + y)2 = 3xy |x + y| 3xy(x + y). (2.2.4) Đẳng thức xảy (2.2.3) (2.2.4) trở thành đẳng thức, tức x, y nghiệm hệ x = y 27x2 y = (x + y)2 Giải hệ ta x = y = x = y = 13 , suy a = b = c = 1, a = b = 43 , c = hoán vị. Bài toán chứng minh. Sau ví dụ đầu ta thấy ý tưởng để giải bất đẳng thức đối xứng bậc “mô hình” hóa sau: (1) Nếu bất đẳng thức có điều kiện a + b + c = ta tiến hành đổi biến sang x = a − 1, y = b − 1, z = c − 1, bất đẳng thức có điều kiện khác ta cố gắng toán thông qua số đánh giá để đưa toán đồng bậc sau chuẩn hóa a + b + c = 3, sử dụng biến đổi a4 + b4 + c4 = x4 + y + z + 4(x3 + y + z ) + 6(x2 + y + z ) + 3, a3 + b3 + c3 = x3 + y + z + 3(x2 + y + z ) + 3, a2 + b2 + c2 = x2 + y + z + 3, ab + bc + ca = xy + yz + zx + 3, abc = xyz + xy + yz + zx + ··· (2) Thay z = −x − y biến đổi hai đại lượng x2 + xy + y , xy (x + y) thông qua đẳng thức x4 + y + z = 2(x2 + xy + y )2 , x3 + y + z = −3xy(x + y), x2 + y + z = 2(x2 + xy + y ), xy + yz + zx = −(x2 + xy + y ), xyz = −xy(x + y), ··· Một số vấn đề bất đẳng thức bậc bốn ba biến Cuối ta thu bất đẳng thức có dạng A(x2 + xy + y ) + B(x2 + xy + y ) + Cxy (x + y) + D 0. Từ tùy vào hệ số A, B, C, D mà ta có đánh giá thích hợp để giải toán. Rất vui mừng phát thú vị thử áp dụng vào toán khác thu kết sau: Bài 2.3. Cho ba số thực a, b, c thỏa mãn a + b + c = 3. Chứng minh (ab + bc + ca − 3)2 27(abc − 1). (2.3.1) Lời giải. Đặt a = x + 1, b = y + 1, c = z + 1, x + y + z = 0. Khi ab + bc + ca = xy + yz + zx + 3. abc = xyz + xy + yz + xz + 1. Bất đẳng thức (2.3.1) trở thành (xy + yz + zx)2 Giả sử xy 27(xyz + xy + yz + xz). (2.3.2) 0, thay z = −x − y vào (2.3.2) thu gọn lại, ta [xy − (x + y)2 ]2 27[−xy(x + y) + xy − (x + y)2 ], (x2 + xy + y )2 + 27(x2 + xy + y ) + 27xy(x + y) 0. Áp dụng bất đẳng thức AM-GM, ta có x2 + xy + y (x + y)2 3xy 0, suy (x2 + xy + y )2 + 27(x2 + xy + y ) 9x2 y + 81 (x + y)2 . Ta chứng minh x2 y + (x + y)2 + 3xy(x + y) bất đẳng thức 0, (3x + 3y + 2xy)2 x2 y + (x + y)2 + 3xy(x + y) = 4 0. Đẳng thức xảy a = b = c = 1, a = b = −2, c = hoán vị. Chứng minh ta hoàn tất. Bài 2.4. Cho ba số thực a, b, c thỏa mãn a + b + c = 3. Chứng minh 3(a4 + b4 + c4 ) + 33 14(a2 + b2 + c2 ). (2.4.1) Một số vấn đề bất đẳng thức bậc bốn ba biến Lời giải. Đặt a = x + 1, b = y + 1, c = z + 1, x + y + z = 0. Bất đẳng thức (2.4.1) viết lại sau 3(x4 + y + z ) + 12(x3 + y + z ) + 4(x2 + y + z ) Giả sử xy 0. (2.4.2) 0, thay z = −x − y vào (2.4.2) thu gọn lại, ta 3(x4 + 2x3 y + 3x2 y + 2xy + y ) + 4(x2 + xy + y ) 3(x2 + xy + y )2 + 4(x2 + xy + y ) 18xy(x + y), 18xy(x + y). Áp dụng bất đẳng thức AM-GM, ta có (x + y)2 x2 + xy + y 3xy 0, suy 3(x2 + xy + y )2 + 4(x2 + xy + y ) 27x2 y + 3(x + y)2 , lại có 27x2 y + 3(x + y)2 − 18xy(x + y) = 3(x + y − 3xy)2 0. Đẳng thức xảy a = b = c = a = b = 35 , c = − 13 . Bài toán chứng minh. Bài 2.5. Cho ba số thực a, b, c thỏa mãn a + b + c = 3. Chứng minh (ab + bc + ca)2 + 18abc. (2.5.1) Lời giải. Đặt a = x + 1, b = y + 1, c = z + 1, x + y + z = 0. Khi bất đẳng thức (2.5.1) trở thành (xy + yz + zx + 3)2 + 18(1 + xy + yz + zx + xyz), hay (xy + yz + zx)2 Giả sử xy 12(xy + yz + zx) + 18xyz. (2.5.2) 0, thay z = −x − y vào (2.5.2), ta (x2 + xy + y )2 + 12(x2 + xy + y ) + 18xy(x + y) 0. Áp dụng bất đẳng thức AM-GM, ta có x2 + xy + y (x + y)2 3xy 0, suy (x2 + xy + y )2 + 12(x2 + xy + y ) 9x2 y + 9(x + y)2 , ta cần x2 y + (x + y)2 + 2xy(x + y) 0, tương đương với (x + y + xy)2 0. Đẳng thức xảy a = b = c = a = b = −1, c = 5. Bài toán chứng minh. Một số vấn đề bất đẳng thức bậc bốn ba biến Bài 2.6. Chứng minh bất đẳng thức (a2 + b2 + c2 − 2ab − 2bc − 2ca)2 + 9(ab + bc + ca)2 30abc(a + b + c), (2.6.1) với số thực dương a, b, c. Lời giải. Nếu a + b + c = (2.6.1) hiển nhiên đúng. Nếu a + b + c = 0, thay (a, b, c) (−a, −b, −c) bất đẳng thức không thay đổi nên ta giả sử a + b + c > 0, chuẩn hóa cho a + b + c = 3. Đặt a = x + 1, b = y + 1, c = z + 1, x + y + z = 0. Bất đẳng thức (2.6.1) trở thành [4(xy + yz + zx) + 3]2 + 9(xy + yz + zx + 3)2 90(xy + yz + zx + xyz), tương đương với 25(xy + yz + zx)2 Giả sử xy 12(xy + yz + zx) + 90xyz. (2.6.2) 0, thay z = −x − y vào (2.6.2) thu gọn lại, ta 25(x2 + xy + y )2 + 12(x2 + xy + y ) + 90xy(x + y) 0. Áp dụng bất đẳng thức AM-GM, ta có x2 + xy + y (x + y)2 3xy 0, suy 25(x2 + xy + y )2 + 12(x2 + xy + y ) 225x2 y + 9(x + y)2 . Ta chứng minh 25x2 y + (x + y)2 + 10xy(x + y) 0, điều 25x2 y + (x + y)2 + 10xy(x + y) = (x + y + 5xy)2 0. Đẳng thức xảy a = b = c a = 3b = 3c hoán vị, chứng minh ta hoàn tất. Bài 2.7. Cho ba số thực a, b, c không âm. Chứng minh 3(a2 + b2 + c2 ) (a + b + c) √ √ √ ab + bc + ca + (b − c)2 (a + b + c)2 . (Việt Nam MO 2005) Lời giải. Thay (a, b, c) (a2 , b2 , c2 ) bất đẳng thức cần chứng minh trở thành 3(a4 + b4 + c4 ) (a2 + b2 + c2 )(ab + bc + ca) + (b2 − c2 )2 (a2 + b2 + c2 )2 . Ta chứng minh vế trái 3(a4 + b4 + c4 ) (a2 + b2 + c2 )(ab + bc + ca) + 2(a4 + b4 + c4 − a2 b2 − b2 c2 − c2 a2 ). Một số vấn đề bất đẳng thức bậc bốn ba biến Điều tương đương với a4 + b4 + c4 + 2(a2 b2 + b2 c2 + c2 a2 ) (a2 + b2 + c2 )2 (a2 + b2 + c2 )(ab + bc + ca), (a2 + b2 + c2 )(ab + bc + ca). (2.7.1) Chuẩn hóa a + b + c = đặt a = x + 1, b = y + 1, c = −x − y + vào (2.7.1) thu gọn lại, ta 6(x4 + 2x3 y + 3x2 y + 2xy + y ) + 9(x2 + xy + y ) 6(x2 + xy + y )2 + 9(x2 + xy + y ) 0, 0. Bất đẳng thức hiển nhiên đúng. Đẳng thức xảy a = b = c, vế trái chứng minh. Tiếp đến ta chứng minh vế bên phải (a2 + b2 + c2 )(ab + bc + ca) + 2(a4 + b4 + c4 − a2 b2 − b2 c2 − c2 a2 ) (a2 + b2 + c2 )2 , bất đẳng thức tương đương a4 + b4 + c4 + (a2 + b2 + c2 )(ab + bc + ca) 4(a2 b2 + b2 c2 + c2 a2 ), a4 + b4 + c4 + abc(a + b + c) + ab(a2 + b2 ) + ca(c2 + a2 ) + ca(c2 + a2 ) 4(a2 b2 + b2 c2 + c2 a2 ). Dễ thấy hệ hai bất đẳng thức sau ab(a2 + b2 ) + ca(c2 + a2 ) + ca(c2 + a2 ) a4 + b4 + c4 + abc(a + b + c) 2(a2 b2 + b2 c2 + c2 a2 ), (2.7.2) ab(a2 + b2 ) + ca(c2 + a2 ) + ca(c2 + a2 ). (2.7.3) Bất đẳng thức (2.7.2) tương đương với ab(a − b)2 + bc(b − c)2 + ca(c − a)2 0. Ta chứng minh (2.7.3), chuẩn hóa cho a+b+c = đặt a = x+1, b = y+1, c = −x−y+1 với xy vào bất đẳng thức thu gọn lại, ta 4(x2 + xy + y )2 + 3(x2 + xy + y ) 18xy(x + y). Theo bất đẳng thức AM-GM ta có 4(x2 + xy + y )2 + 3(x2 + xy + y ) · (3xy)2 + · (x + y)2 = 36x2 y + (x + y)2 , 9(x + y − 4xy)2 36x2 y + (x + y)2 − 18xy(x + y) = 0. 4 Đẳng thức xảy a = b = c a = b, c = 0. Như toán chứng minh. Bài 2.8. Cho ba số thực a, b, c thỏa mãn abc = −1, chứng minh a4 + b4 + c4 + 3(a + b + c) a2 + b b + c c + a2 + + . c a b (Iran MO 2005) Một số vấn đề bất đẳng thức bậc bốn ba biến Lời giải. Ta viết bất đẳng thức lại dạng sau a4 + b4 + c4 − 3abc(a + b + c) −ab(a2 + b2 ) − bc(b2 + b2 ) − ca(c2 + a2 ). (2.8.1) Vì ab(a2 + b2 ) = (ab + bc + ca)(a2 + b2 + c2 ) − abc(a + b + c), nên (2.8.1) tương đương với a4 + b4 + c4 + (ab + bc + ca)(a2 + b2 + c2 ) 4abc(a + b + c). (2.8.2) Nếu a + b + c = 0, thay c = −a − b vào (2.8.2) ta a4 + b4 + (a + b)4 − (a2 + ab + b2 ) a2 + b2 + (a + b)2 0, nhiên dễ thấy a4 + b4 + (a + b)4 − (a2 + ab + b2 ) a2 + b2 + (a + b)2 = 0, nên toán trường hợp này. Nếu a + b + c = 0, thay (a, b, c) (−a, −b, −c) bất đẳng thức không thay đổi nên ta giả sử a + b + c > 0, chuẩn hóa cho a + b + c = 3. Đặt a = x + 1, b = y + c = −x − y + 1, thay vào (2.8.2) thu gọn lại ta 27(x2 + xy + y ) 0. Bất đẳng thức đúng. Đẳng thức xảy a = b = c a + b + c = 0. Bài toán chứng minh. Nhận xét. Bài toán hệ đẳng thức sau a4 + bc a2 = (a + b + c)2 (a2 + b2 + c2 − ab − bc − ca) + 4abc(a + b + c). Bài 2.9. Chứng minh với số thực a, b, c ta có (a + b)4 + (b + c)4 + (c + a)4 4 a + b4 + c4 + (a + b + c)4 . (2.9.1) Lời giải. Nếu a + b + c = 0, thay c = −a − b vào (2.9.1) thu gọn ta a4 + 2a3 b + 3a2 b2 + 2ab3 + b4 0, ta có a4 + 2a3 b + 3a2 b2 + 2ab3 + b4 = (a2 + ab + b2 )2 0. Nếu a + b + c = 0, thay (a, b, c) (−a, −b, −c) bất đẳng thức không thay đổi nên ta giả sử a + b + c > 0, chuẩn hóa cho a + b + c = 3. Đặt a = x + 1, b = y + 1, c = z + 1, x + y + z = x3 + y + z = 3xyz, ta có (a + b)4 + (b + c)4 + (c + a)4 = x4 + y + z + 24(x2 + y + z ) − 24xyz + 48, Một số vấn đề bất đẳng thức bậc bốn ba biến tương tự a4 + b4 + c4 + (a + b + c)4 = x4 + y + z + 6(x2 + y + z ) + 12xyz + 84. Như bất đẳng thức trở thành x4 + y + z + 48(x2 + y + z ) Giả sử xy 72xyz. (2.9.2) thay z = −x − y vào (2.9.2), bất đẳng thức viết lại dạng 2(x2 + xy + y )2 + 96(x2 + xy + y ) + 72xy(x + y) 0. Theo bất đẳng thức AM-GM, ta có 2(x2 + xy + y )2 + 96(x2 + xy + y ) 18x2 y + 72(x + y)2 , lại có 18x2 y + 72(x + y)2 + 72xy(x + y) = 18(2x + 2y + xy)2 0. Đẳng thức xảy a = b = c −3a = −3b = c. Chứng minh hoàn tất. Nhận xét. Bài toán kết mạnh bất đẳng thức sau 4 (a + b4 + c4 ). (a + b)4 + (b + c)4 + (c + a)4 (Việt Nam TST 1996) Bài 2.10. Với ba số thực a, b, c. Chứng minh với số thực k ta có (a − b)(a − c)(a − kb)(a − kc) 0. (2.10.1) Lời giải. Lập luận giả sử a + b + c > 0, chuẩn hóa cho a + b + c = 3. Đặt a = x + 1, b = y + 1, c = −x − y + với xy 0, bất đẳng thức (2.10.1) viết lại dạng (k + 2)2 (x2 + xy + y )2 + 3(k − 1)2 (x2 + xy + y ) 9(k + 2)(1 − k)xy(x + y). (2.10.2) Theo bất đẳng thức AM-GM, ta có x2 + xy + y (x + y)2 3xy 0. Suy (k + 2)2 (x2 + xy + y )2 + 3(k − 1)2 (x2 + xy + y ) 9(k + 2)2 x2 y + (k − 1)2 (x + y)2 , Ta chứng minh (k + 2)2 x2 y + (k − 1)2 (x + y)2 (k + 2)(1 − k)xy(x + y), tương đương với [(k − 1)(x + y) + 2(k + 2)xy]2 0. Đẳng thức xảy a = b = c a = kb = kc. Chứng minh hoàn tất. 10 Một số vấn đề bất đẳng thức bậc bốn ba biến Nhận xét. Bài toán hệ đẳng thức (a − b)(a − c)(a − kb)(a − kc) = (a − b)2 [a + b − (k + 1)c]2 . Đây kết thú vị, có số kết đẹp mắt sau Nếu k = 0, bất đẳng thức trở thành a2 (a − b)(a − c) + b2 (b − c)(b − a) + c2 (c − a)(c − b) 0, bất đẳng thức Schur bậc khai triển ta bất đẳng thức (2.7.3). Nếu k = 2, bất đẳng thức trở thành 9(a4 + b4 + c4 ) + 126(a2 b2 + b2 c2 + c2 a2 ) Đẳng thức xảy a 5(a + b + c)4 . = b = c. 3. Đưa toán dạng bậc bốn Có nhiều toán dạng phân thức, thức, . nhìn dạng đa thức bậc thông qua số đánh giá biến đổi, ta chuyển chúng dạng đa thức bậc áp dụng phương pháp trên. Để hiểu rõ hơn, mời ban đọc xét qua toán sau. Bài 3.1. Cho a, b, c số thực không âm thỏa mãn ab + bc + ca > 0. Chứng minh bất đẳng thức sau (a + b)2 (b + c)2 (c + a)2 + + c2 + ab a + bc b + ca 6. Lời giải. Áp dụng bất đẳng thức Cauchy-Schwarz ta có (a + b)2 c2 + ab [(a + b)2 + (b + c)2 + (c + a)2 ]2 (ab + c2 )(a + b)2 + (bc + a2 )(b + c)2 + (ca + b2 )(c + a + b)2 4(a2 + b2 + c2 + ab + bc + ca)2 = . (ab + c2 )(a + b)2 + (bc + a2 )(b + c)2 + (ca + b2 )(c + a + b)2 Vì để chứng minh toán ta cần chứng minh (a2 + b2 + c2 + ab + bc + ca)2 (ab + c2 )(a + b)2 + (bc + a2 )(b + c)2 + (ca + b2 )(c + a)2 . Bằng cách khai triển trực tiếp ta thấy điều tương đương với 2[a4 + b4 + c4 + abc(a + b + c) − ab(a2 + b2 )] + ab(a − b)2 0. Bất đẳng thức cuối theo bất đẳng thức (2.7.3), đẳng thức xảy a = b = c a = b, c = hoán vị. Bài toán chứng minh. Bài 3.2. Cho a, b, c số thực không âm thỏa mãn a2 + b2 + c2 + ab + bc + ca = 6. Chứng minh 1 + + (3.2.1). − ab − bc − ca 11 Một số vấn đề bất đẳng thức bậc bốn ba biến Lời giải. Bài toán làm √ chặt từ đề thi Moldova TST 2005 có đến hai dấu a = b = c = a = b = 2, c = 0. Muốn sử dụng bất đẳng thức Cauchy-Schwarz ta phải dùng đến kỹ thuật thêm bớt để làm đảo chiều toán. Ta xét P (a, b) = k − . − ab Ta có P (1, 1) = k − √ P 2, = k − √ √ P 2, = k − , , . Ta cần tìm k cho P (a, b) đánh giá chặt tốt từ tính toán ta thấy k giá trị tốt cần tìm, từ dẫn đến đẳng thức 1− − ab = , − ab − ab bất đẳng thức (3.2.1) viết lại sau − ab − bc − bc + + − ab − bc − bc 1. Do a2 + b2 + c2 + ab + bc + ca = 6, nên 3(2 − ab) = (a − b)2 + c2 + bc + ca 0. Theo bất đẳng thức Cauchy-Schwarz, ta có (6 − ab − bc − ca)2 [ (2 − ab)]2 = = (2 − ab)(4 − ab) (2 − ab)(4 − ab) − ab − ab (a2 + b2 + c2 )2 . (2 − ab)(4 − ab) Như vậy, ta cần chứng minh (a2 + b2 + c2 )2 (2 − ab)(4 − ab), tương đương với a4 + b4 + c4 + a2 b2 + b2 c2 + c2 a2 + 6(ab + bc + ca) 24, hay dạng a4 + a2 b2 + ab a2 + bc a2 + bc . Khi triển thu gọn lại ta a4 + b4 + c4 + abc(a + b + c) ab(a2 + b2 ) + bc(b2 + c2 ) + ca(c2 + a2 ). Đây bất đẳng thức (2.7.3). Bài toán chứng minh. 12 Một số vấn đề bất đẳng thức bậc bốn ba biến Bài 3.3. Cho ba số thực x, y, z thuộc [−1, 1] thỏa mãn x + y + z = 0. Chứng minh √ + x + y + + y + z + + z + x2 3. Lời giải. Bình phương hai vế, ta x2 + y + z + (1 + x + y )(1 + y + z ) 6. Áp dụng bất đẳng thức Cauchy-Schwarz ta có (1 + x + y )(1 + y + z ) (1 + x)(1 + y) + |yz| (1 + x)(1 + y) − yz. Ta đưa toán chứng minh x2 − yz + (1 + x)(1 + y) 6. Đặt 1+x = a2 , 1+y = b2 , 1+z = c2 với a, b, c số không âm, a2 +b2 +c2 = 3, bất đẳng thức trở thành a4 − b2 c + a2 + bc − 0, tương đương với a4 − a4 − b2 c + b2 c + a2 bc − bc − a2 0, a2 0, khai triển thu gọn lại, ta a4 + b4 + c4 + abc(a + b + c) − ab(a2 + b2 ) + bc(b − c)2 0. Bất đẳng thức theo bất đẳng thức (2.7.3), đẳng thức xảy x = y = z = 0. Bài toán chứng minh. Bài 3.4. Cho ba số thực không âm a, b, c thỏa mãn a2 + b2 + c2 > 0. Chứng minh √ √ abc(ab + bc + ca) 2 a +b +c + 3· √ 2(ab + bc + ca). a2 + b + c Lời giải. Áp dụng bất đẳng thức AM-GM ta có √ √ √ √ √ abc(ab + bc + ca) 3abc 3abc a2 + b2 + c2 √ 3· √ = a2 + b + c a2 + b + c a2 + b2 + c2 3abc(a + b + c) . a2 + b + c Ta cần chứng minh a2 + b + c + 3abc(a + b + c) a2 + b + c 2(ab + bc + ca), quy đồng thu gọn lại ta a4 + b4 + c4 + abc(a + b + c) ab(a2 + b2 ) + bc(b2 + c2 ) + ca(c2 + a2 ). Đây bất đẳng thức (2.7.3), đẳng thức xảy a = b = c a = b, c = hoán vị. Chứng minh hoàn tất. 13 Một số vấn đề bất đẳng thức bậc bốn ba biến Bài 3.5. Cho ba số thực không âm a, b, c thỏa mãn a3 + b3 + c3 + abc = 12. Chứng minh bất đẳng thức sau thỏa mãn 19(a2 + b2 + c2 ) + 6(ab + bc + ca) a+b+c 36. Lời giải. Áp dụng bất đẳng thức AM-GM, ta có 19(a2 + b2 + c2 ) + 6(ab + bc + ca) = 8(a2 + b2 + c2 ) + 8(a2 + b2 + c2 ) + 3(a + b + c)2 12 3(a2 + b2 + c2 )2 (a + b + c)2 . Suy 19(a2 + b2 + c2 ) + 6(ab + bc + ca) a+b+c 12 3(a2 + b2 + c2 )2 . a+b+c Ta quy toán chứng minh (a2 + b2 + c2 )2 9(a + b + c), hay 4(a2 + b2 + c2 )2 (a3 + b3 + c3 + abc)(a + b + c). Chuẩn hóa a + b + c = đặt a = x + 1, b = y + 1, c = −x − y + 1, với xy thức cần chứng minh trở thành 16(x2 + xy + y )2 + 3(x2 + xy + y ) + 36xy(x + y) bất đẳng 0. Theo bất đẳng thức AM-GM 16(x2 + xy + y )2 + 3(x2 + xy + y ) 144x2 y + (x + y)2 , lại có 9(x + y + 8xy)2 144x y + (x + y) + 36xy(x + y) = 0. 4 Đẳng thức xảy a = 2b = 2c hoán vị. Bài toán chứng minh. 2 4. Xung quanh toán kỳ thi British MO 1986 Trong kỳ thi vô địch toán British MO 1986 có toán bất đẳng thức thú vị sau Bài 4.1. Với a, b, c ba số thực thỏa mãn đồng thời điều kiện a + b + c = 0, a2 + b2 + c2 = 6. Chứng minh a2 b + b c + c a 6. Đây bất đẳng thức khó đẳng thức không tâm, không biên mà a = cos 2π , b = cos 4π , c = cos 8π hoán vị, dấu “kỳ lạ” 9 mà toán gây khó khăn cho phương pháp mà ta biết phương pháp mạnh S.O.S, dồn biến, . Trong sách “Sử Dụng Phương Pháp 14 Một số vấn đề bất đẳng thức bậc bốn ba biến Cauchy Schwarz Để Chứng Minh Bất Đẳng Thức” hai tác giả Võ Quốc Bá Cẩn, Trần Quốc Anh đưa chứng minh độc đáo bất đẳng thức Cauchy-Schwarz sau Lời giải. Ta có số tính toán đơn giản ab + bc + ca = (a + b + c)2 − (a2 + b2 + c2 ) = −3, a2 b2 + b2 c2 + c2 a2 = (ab + bc + ca)2 − 2abc(a + b + c) = 9. Sử dụng đẳng thức 3(a2 b + b2 c + c2 a) = a(2ab + c2 ) + b(2bc + a2 ) + c(2ca + b2 ), (4.1.1) bất đẳng thức Cauchy-Schwarz, ta có (a2 b + b2 c + c2 a)2 (a2 + b2 + c2 ) [(2ab + c2 )2 + (2bc + a2 )2 + (2ca + b2 )2 ] [(2ab + c2 )2 + (2bc + a2 )2 + (2ca + b2 )2 ] = . Bằng cách khai triển trực tiếp, ta thấy (2ab + c2 ) = (a2 + b2 + c2 )2 + 2(a2 b2 + b2 c2 + b2 c2 ) + 4abc(a + b + c) = 54, nên a2 b + b c + c a 6, suy a2 b + b c + c a 6. Bài toán chứng minh. Nhận xét. Có thể nói mấu chốt lời giải việc sử dụng đẳng thức (4.1.1) để sau sử dụng thành công bất đẳng thức Cauchy-Schwarz. Trong sách tác giả lý giải việc tìm đẳng thúc (4.1.1) cách sử dụng phương pháp nhân tử Langrange (sẽ học chương trình toán cao cấp bậc đại học) sau. Bằng cách đặt F (a, b, c) = a2 b + b2 c + c2 a + λ1 (a + b + c) + λ2 (a2 + b2 + c2 − 6). Ta thấy điểm cực trị hàm F (a, b, c) nghiệm hệ phương trình sau ∂F ∂F ∂F ∂a = ∂b = ∂c = a+b+c=0 a2 + b2 + c2 = tương đương với 2ab + c2 + λ1 + 2λ2 a = 2bc + a + λ1 + 2λ2 b = 2ca + b2 + λ1 + 2λ2 c = a + b + c = a + b2 + c2 = 6 (4.1.2) Hà Nội 15 Một số vấn đề bất đẳng thức bậc bốn ba biến Cộng tương ứng theo vế phương trình thứ nhất, thứ hai, thứ ba lại với ta (a + b + c)2 + 3λ1 + 2λ2 (a + b + c) = 0, Từ suy λ1 = 0, hệ (4.1.2) lúc trở thành 2ab + c2 + 2λ2 a = 2bc + a + 2λ2 b = 2ca + b2 + 2λ2 c = a+b+c=0 a + b2 + c2 = Từ ba phương trình đầu, ta rút 2bc + a2 2ca + b2 2ab + c2 = = = −2λ2 . a b c Nếu để ý ta thấy đẳng thức điều kiện xảy dấu bất đẳng thức Cauchy-Schwarz điều lý giải cho việc tách đẳng thức (4.1.1). Không dừng lại đây, kỹ thuật tương tự ta giải trọn vẹn toán tổng quát bất đẳng thức trên, toán hay. Bài 4.2. Với a, b, c ba số thực t số cho trước thỏa mãn đồng thời a + b + c = 0, a2 + b2 + c2 = 6t2 . Chứng minh với số thực k ta có bất đẳng thức sau √ a2 b + b2 c + c2 a + kabc 2t3 k − 3k + 9. (4.2.1) Lời giải. Từ giả thiết ta tính ab + bc + ca = −3t2 , a2 b2 + b2 c2 + c2 a2 = 9t4 . Lại có a3 b + b3 c + c3 a − ab3 − bc3 − ca3 = −(a + b + c)(a − b)(b − c)(c − a) = 0, nên a3 b + b3 c + c3 a = ab3 + bc3 + ca3 , suy ab(a2 + b2 ) + bc(b2 + c2 ) + ca(c2 + a2 ) 2 2 ab(a + b ) + bc(b + c2 ) + ca(c2 + a2 ) + abc(a + b + c) = (ab + bc + ca)(a2 + b2 + c2 ) = = −9t4 . Từ áp dụng bất đẳng thức Cauchy-Schwarz, ta có a3 b + b c + c a = 3(a2 b + b2 c + c2 a + kabc) = 3(a2 b + b2 c + c2 a) + 3kabc + kt2 (a + b + c) a(2ab + c2 + kbc + kt2 ) = (a2 + b2 + c2 ) = 6t2 (2ab + c2 + kbc + kt2 ) (2ab + c2 + kbc + kt2 ) . 16 Một số vấn đề bất đẳng thức bậc bốn ba biến Bằng khai triển trực tiếp ta thấy (2ab + c2 + kbc + kt2 ) = 6t4 (k − 3k + 9). Suy a2 b + b2 c + c2 a + kabc √ 2t3 k − 3k + 9. Bài toán chứng minh. Nhận xét. (1) Ngoài để tính a3 b + b3 c + c3 a = −9t4 ta sử dụng đẳng thức (a + b + c)(a2 b + b2 c + c2 a) = 0. (2) Chọn k = bất đẳng thức (4.2.1) trở thành 2(a2 b + b2 c + c2 a) + 3abc √ 3t3 . (4.2.2) Do a + b + c = nên ta dễ dàng kiểm tra 2(a2 b + b2 c + c2 a) + 3abc = |(a − b)(b − c)(c − a)| , bất đẳng thức (4.2.1) tương đương với |(a − b)(b − c)(c − a)| √ 3t3 . (4.2.3) Lúc chọn tiếp t = 1, ta toán đẹp sau đây: 17 Một số vấn đề bất đẳng thức bậc bốn ba biến Với a, b, c ba số thực cho trước thỏa mãn đồng thời a + b + c = 0, a2 + b2 + c2 = 6. Chứng minh bất đẳng thức √ |(a − b)(b − c)(c − a)| 3. (JBMO Turkey 2014) Những toán bất đẳng thức khó, hình thức đơn giản, đẹp mắt có dấu đẳng thức xảy biến lệch (lệch tâm lệch biên) thường nhiều, để xây dựng toán đòi hỏi người đề phải có trình độ lão luyện. Chắc hẳn đôi lần chạm trán với toán vậy. Có thể nêu toán đại diện cho tiêu chuẩn nói trên, bất đẳng thức tiếng giáo sư Vasile Cirtoaje. Bài 4.3. Cho ba số thực a, b, c thay đổi bất kỳ. Chứng minh (a2 + b2 + c2 )2 3(a3 b + b3 c + c3 a). Ẩn bên vẻ bề đơn giản bất đẳng thức vô khó trường hợp tầm thường a = b = c để đẳng thức xảy trường hợp đặc biệt (a, b, c) ∼ sin2 π7 , sin2 2π , sin2 4π , mà lời giải bình thường cho 7 bất đẳng thức có dấu “bất thường” dường ! Những chứng minh ban đầu đa phần đưa toán dạng tổng bình phương (ngay phương pháp tam thức bậc hai buộc ta phải phân tích biệt thức ∆ thành tổng bình phương) điều đòi hỏi người làm toán phải có nhãn quan nhạy bén để nhận biết tồn đại lượng bình phương đó. Bằng toán tổng quát có lời giải thú vị sau đây. Lời giải. Tương tự (2.9) ta chuẩn hóa a + b + c = 3, tồn số thực t cho a2 + b2 + c2 = + 6t2 . Tiếp đến đặt a = + x, b = + y, c = + z x + y + z = tính x2 + y + z = 6t2 . Bất đẳng thức cần chứng minh trở thành 3(1 + 2t2 ) Trong (4.2.1) chọn k = 1, với ý |A| a3 b + b3 c + c3 a. A, ta a2 b + b2 c + c2 a + abc √ · t3 . Biến đổi vế trái a3 b + b3 c + c3 a = (1 + x)3 (1 + y) + (1 + y)3 (1 + z) + (1 + z)3 (1 + x) = (x3 + 3x2 y + 3xy + x3 y + 3x2 + 1) = x3 + y + z + 3(x2 y + y z + z x) + x2 + yz + x3 y + = 3xyz + 3(x2 y + y z + z x) − 9t4 + 9t2 + = 3(x2 y + y z + z x + xyz) − 9t4 + 9t2 + √ · · t3 − 9t4 + 9t2 + 3. 18 Một số vấn đề bất đẳng thức bậc bốn ba biến Như ta cần chứng minh √ 2t3 − 3t4 + 3t2 + 1, (1 + 2t2 ) √ 7t3 . 7t4 + t2 Áp dụng bất đẳng thức AM-GM ta có √ √ 7t4 · t2 = 7t3 . 7t4 + t2 Bài toán chứng minh. Nhận xét. Với a + b + c = số tốt (về mặt tính toán) để a2 + b2 + c2 = + 6t2 . Thật vậy, theo bất đẳng thức Cauchy-Schwarz ta có (a + b + c)2 = 3, a2 + b2 + c2 nên tồn số thực k không âm t < cho a2 + b2 + c2 = + kt2 . Ta chọn k tùy ý k = 1, k = 10, . . . ta phải chọn k cho đánh giá trở phải nên đơn giản nhất. Cũng theo bất đẳng thức Cauchy-Schwarz a2 + b2 + c2 a2 + (3 − a)2 (b + c)2 = a2 + , 2 suy + kt2 a2 + (3 − a)2 . Giải bất phương trình này, ta √ − t 6k a √ + t 6k , − 2t a + 2t. cho k = 6, Như k = số tốt nhất. Bài 4.4. Tìm số M nhỏ cho bất đẳng thức ab(a2 − b2 ) + bc(b2 − c2 ) + ca(c2 − a2 ) M (a2 + b2 + c2 )2 , với số thực a, b, c bất kỳ. (IMO 2006) √ √ √ , 32 Lời giải. Cho a = 22+2 , b = 2−32 , c = M giá trị nhỏ cần tìm, tức chứng minh ab(a − b ) + bc(b − c ) + ca(c − a ) ta chứng minh M = √ 32 √ 2 (a + b2 + c2 )2 . 32 Thật bất đẳng thức tương đương với |(a + b + c)(a − b)(b − c)(c − a)| √ 2 (a + b2 + c2 )2 . 32 (4.4.1) 19 Một số vấn đề bất đẳng thức bậc bốn ba biến để a2 + b2 + c2 = + 6t2 , lúc Chuẩn hóa a + b + c = 3, tồn số thực t (4.4.1) viết lại dạng √ 27 (1 + 2t2 )2 . 32 |(a − b)(b − c)(c − a)| (4.4.2) Đặt a = x + 1, b = y + 1, c = z + thay vào (4.4.2) ta √ 27 |(x − y)(y − z)(z − x)| (1 + 2t2 )2 . 32 Áp dụng bất đẳng thức (4.2.3) ta đưa toán chứng minh bất đẳng thức mạnh √ √ 27 (1 + 2t2 )2 , 3·t 32 hay √ 32 2 (1 + 2t ) ·t . Áp dụng bất đẳng thức AM-GM, ta có 2 + 2t = + · t2 + · t2 + · t2 3 bình phương hai vế ta 2 (1 + 2t ) Điều cho phép ta kết luận M = √ , 32 1· 2 ·t =4 ·t , 27 √ 32 ·t . giá trị nhỏ cần tìm. Không áp dụng với lớp toán bậc 4, bổ đề tổng quát áp dụng tốt với toán khác, xem Bài 4.5. Với a, b, c ba số thực thỏa mãn đồng thời điều kiện a + b + c = 0, a2 + b2 + c2 = 3. Chứng minh a5 b + b c + c a −3. Lời giải. Do a2 + b2 + c2 = nên từ (4.2) ta t = √1 . Ta có có số tính toán ab + bc + ca = − , 3 a b+b c+c a=− . a2 b + b2 c + c2 a + 3abc = −(ab2 + bc2 + ca2 ) a3 b3 + b3 c3 + c3 a3 = (ab + bc + ca)3 + 3a2 b2 c2 = − 27 + 3a2 b2 c2 . 20 Một số vấn đề bất đẳng thức bậc bốn ba biến Ta có biến đổi a5 b = a3 b · a2 = a3 b(3 − b2 − c2 ) = 3a3 b − a3 b3 − a3 bc2 , suy a5 b + b5 c + c5 a = 3(a3 b + b3 c + c3 a) − (a3 b3 + b3 c3 + c3 a3 ) − (ab2 + bc2 + ca2 )abc 27 =3· − − − + 3a2 b2 c2 + (a2 b + b2 c + c2 a + 3abc)abc 27 = − + abc(a2 b + b2 c + c2 a). Bài toán quy chứng minh . abc(a2 b + b2 c + c2 a) Trong (4.2.1) chọn k = 3, t = √12 , ta √ . a2 b + b2 c + c2 a + 3abc (4.6.1) Áp dụng bất đẳng thức AM-GM, ta có · 3abc · (a2 b + b2 c + c2 a) 3abc + a2 b + b2 c + c2 a 3 . √ 2π √ 13π √ 7π cos , cos Đẳng thức xảy (a, b, c) = , cos 9 √ √ √ 2π 13π 7π , − cos hoán vị. (a, b, c) = − cos , − cos 9 abc(a2 b + b2 c + c2 a) = , Bài toán chứng minh. Nhận xét. Chắc hẳn bạn đọc thắc mắc ta lại chọn k = để có đánh giá (4.6.1). Ta lý giải điều sau: Do t = √1 , nên từ (4.2.1) ta a2 b + b2 c + c2 a + kabc k − 3k + . Áp dụng bất đẳng thức AM-GM ta có abc(a2 b + b2 c + c2 a) · kabc · (a2 b + b2 c + c2 a) k kabc + a2 b + b2 c + c2 a k 2 k − 3k + . |k| Như ta cần tìm k cho k − 3k + = |k| . Giải phương trình ta k = 3. 21 Một số vấn đề bất đẳng thức bậc bốn ba biến 5. Bài tập rèn luyện Bài 5.1. Cho a, b, c ba số thực không âm cho ab + bc + ca > 0. Chứng minh a2 b2 c2 + + 2a2 + (b + c)2 2b2 + (c + a)2 2c2 + (a + b)2 . Bài 5.2. Chứng minh bất đẳng thức a2 + b2 + c2 + (a + b + c)2 12 a4 + b4 + c4 + (a + b + c)4 , với số thực a, b, c thay đổi bất kỳ. Bài 5.3. Cho a, b, c số thực không âm.Chứng minh √ a4 + b4 + c4 − abc(a + b + c) 2 a3 b + b3 c + c3 a − ab3 − bc3 − ca3 . Bài 5.4. Cho ba số thực x, y, z có tổng 3. Chứng minh 4(x4 + y + z ) + 45 19(x2 + y + z ). Bài 5.5. Cho a, b, c độ dài ba cạnh tam giác. Chứng minh a4 + b4 + c4 + 9abc(a + b + c) 10(a2 b2 + b2 c2 + c2 a2 ). Bài 5.6. Cho a, b, c ba số thực thỏa mãn a2 + b2 + c2 = 3. Tìm giá trị lớn P = a4 + b4 + c4 + 3(ab + bc + ca). Bài 5.7. Cho a, b, c ba số thực thỏa mãn ab + bc + ca = 3. Tìm giá trị nhỏ F = 4(a4 + b4 + c4 ) + 11abc(a + b + c). Bài 5.8. Tìm số k lớn cho bất đẳng a4 + b + c + k(a3 + b3 + c3 + 3abc − 6), với số thực dương a, b, c thỏa mãn a + b + c = 3. Bài 5.9. Chứng minh với số thực không âm a, b, c a4 + b + c − a2 b − b c − c a2 a3 b + b3 c + c3 a − ab3 − bc3 − ca3 . Bài 5.10. Cho ba số thực α, β, γ thỏa mãn + α + β = 2γ. Chứng minh bất đẳng thức a4 + α b2 c2 + βabc a γ bc(b2 + c2 ), với số thực a, b, c + α γ 2. 22 TÀI LIỆU THAM KHẢO [1] Võ Quốc Bá Cẩn, Chuyên Đề Bất Đẳng Thức Hiện Đại, 2008. [2] Võ Quốc Bá Cẩn, Trần Quốc Anh, Sử Dụng Phương Pháp Cauchy Schwarz Để Chứng Minh Bất Đẳng Thức, Nhà Xuất Bản Đại Học Sư Phạm, 2010. [3] Lê Việt Hải, Phương Pháp Nhân Tử Langrange & Bất Đẳng Thức Cauchy-Schwarz, Câu Lạc Bộ Toán Trường Phổ Thông Năng Khiếu Thành Phố Hồ Chí Minh, 2011. [4] Vasile Cirtoaje, Algebraic Inequalities Polynomial Rational Symmetric Inequalities, GIL Publishing House, 2011. [5] Vasile Cirtoaje, Võ Quốc Bá Cẩn , On Some Cyclic Homogeneous Polynomial Inequality Of Degree Fourth In Real Variables Under Constraints, International Journal of Pure and Applied Mathematics, 2012. [6] Art of Problem Solving: http://artofproblemsolving.com 23 [...]... vấn đề về bất đẳng thức bậc bốn ba biến Nhận xét Bài toán là hệ quả của đẳng thức 2 (a − b)(a − c)(a − kb)(a − kc) = (a − b)2 [a + b − (k + 1)c]2 Đây là một kết quả thú vị, chúng ta có một số kết quả khá đẹp mắt như sau Nếu k = 0, bất đẳng thức trở thành a2 (a − b)(a − c) + b2 (b − c)(b − a) + c2 (c − a)(c − b) 0, đây chính là bất đẳng thức Schur bậc 4 và nếu khai triển ta sẽ được bất đẳng thức (2.7.3)... tiếp t = 1, ta sẽ được bài toán rất đẹp sau đây: 17 Một số vấn đề về bất đẳng thức bậc bốn ba biến Với a, b, c là ba số thực cho trước thỏa mãn đồng thời a + b + c = 0, a2 + b2 + c2 = 6 Chứng minh bất đẳng thức √ |(a − b)(b − c)(c − a)| 6 3 (JBMO Turkey 2014) Những bài toán bất đẳng thức khó, hình thức đơn giản, đẹp mắt và có dấu đẳng thức xảy ra khi các biến lệch nhau (lệch tâm và lệch biên) thường không... với a, b, c là các số không âm, khi đó a2 +b2 +c2 = 3, bất đẳng thức trên trở thành a4 − 2 b2 c 2 + 2 a2 + 2 bc − 9 0, tương đương với a4 − 2 3 a4 − 6 b2 c 2 + 2 b2 c 2 + 2 a2 bc − bc − a2 0, a2 0, khai triển và thu gọn lại, ta được a4 + b4 + c4 + abc(a + b + c) − ab(a2 + b2 ) + 2 bc(b − c)2 0 Bất đẳng thức này đúng theo bất đẳng thức (2.7.3), đẳng thức xảy ra khi x = y = z = 0 Bài toán được chứng minh... Một số vấn đề về bất đẳng thức bậc bốn ba biến Cauchy Schwarz Để Chứng Minh Bất Đẳng Thức hai tác giả Võ Quốc Bá Cẩn, Trần Quốc Anh 6 đã đưa ra một chứng minh rất độc đáo bằng bất đẳng thức Cauchy-Schwarz như sau Lời giải Ta có một số tính toán đơn giản ab + bc + ca = (a + b + c)2 − (a2 + b2 + c2 ) = −3, 2 a2 b2 + b2 c2 + c2 a2 = (ab + bc + ca)2 − 2abc(a + b + c) = 9 Sử dụng đẳng thức 3(a2 b + b2... a2 2ca + b2 2ab + c2 = = = −2λ2 a b c Nếu để ý ta sẽ thấy đẳng thức trên chính là điều kiện xảy ra dấu bằng trong bất đẳng thức Cauchy-Schwarz và điều này lý giải cho việc tách đẳng thức (4.1.1) Không dừng lại ở đây, bằng kỹ thuật tương tự chúng ta ta có thể giải quyết trọn vẹn được bài toán tổng quát của bất đẳng thức trên, một bài toán rất hay Bài 4.2 Với a, b, c là ba số thực và t 0 là một số cho... c) − ab(a2 + b2 )] + 3 ab(a − b)2 0 Bất đẳng thức cuối cùng đúng theo bất đẳng thức (2.7.3), đẳng thức xảy ra khi và chỉ khi a = b = c hoặc a = b, c = 0 và các hoán vị Bài toán được chứng minh Bài 3.2 Cho a, b, c là các số thực không âm thỏa mãn a2 + b2 + c2 + ab + bc + ca = 6 Chứng minh rằng 1 1 1 + + 1 (3.2.1) 4 − ab 4 − bc 4 − ca 11 Một số vấn đề về bất đẳng thức bậc bốn ba biến Lời giải Bài toán... đây một bài toán đại diện cho những tiêu chuẩn nói trên, một bất đẳng thức rất nổi tiếng của giáo sư Vasile Cirtoaje Bài 4.3 Cho ba số thực a, b, c thay đổi bất kỳ Chứng minh rằng (a2 + b2 + c2 )2 3(a3 b + b3 c + c3 a) Ẩn bên trong vẻ bề ngoài đơn giản này là một bất đẳng thức vô cùng khó vì ngoài trường hợp tầm thường a = b = c để đẳng thức xảy ra thì vẫn còn một trường hợp nữa đặc biệt nữa là (a,... 9t4 + 9t2 + 3 18 Một số vấn đề về bất đẳng thức bậc bốn ba biến Như vậy ta chỉ cần chứng minh √ 2t3 7 − 3t4 + 3t2 + 1, 2 (1 + 2t2 ) hay là √ 2 7t3 7t4 + t2 Áp dụng bất đẳng thức AM-GM ta có √ √ 2 7t4 · t2 = 2 7t3 7t4 + t2 Bài toán được chứng minh Nhận xét Với a + b + c = 3 thì 6 là hằng số tốt nhất (về mặt tính toán) để a2 + b2 + c2 = 3 + 6t2 Thật vậy, theo bất đẳng thức Cauchy-Schwarz ta có (a + b... Đặt a = x + 1, b = y + 1, c = z + 1 rồi thay vào (4.4.2) ta được √ 27 2 |(x − y)(y − z)(z − x)| (1 + 2t2 )2 32 Áp dụng bất đẳng thức (4.2.3) ta đưa bài toán về chứng minh bất đẳng thức mạnh hơn là √ √ 27 2 3 (1 + 2t2 )2 , 6 3·t 32 hay √ 32 6 3 2 2 (1 + 2t ) ·t 9 Áp dụng bất đẳng thức AM-GM, ta có 2 2 2 1 + 2t = 1 + · t2 + · t2 + · t2 3 3 3 2 bình phương hai vế ta được 2 2 (1 + 2t ) Điều này cho phép... khi a = b = c hoặc a = b, c = 0 cùng các hoán vị Chứng minh hoàn tất 13 Một số vấn đề về bất đẳng thức bậc bốn ba biến Bài 3.5 Cho ba số thực không âm a, b, c thỏa mãn a3 + b3 + c3 + abc = 12 Chứng minh rằng bất đẳng thức sau luôn được thỏa mãn 19(a2 + b2 + c2 ) + 6(ab + bc + ca) a+b+c 36 Lời giải Áp dụng bất đẳng thức AM-GM, ta có 19(a2 + b2 + c2 ) + 6(ab + bc + ca) = 8(a2 + b2 + c2 ) + 8(a2 + b2 + . 0, bất đẳng thức trở thành a 2 (a −b)(a − c) + b 2 (b −c)(b − a) + c 2 (c −a)(c − b) 0, đây chính là bất đẳng thức Schur bậc 4 và nếu khai triển ta sẽ được bất đẳng thức (2.7.3). Nếu k = 2, bất. bất đẳng thức đối xứng bậc 4 được “mô hình” hóa như sau: (1) Nếu bất đẳng thức có điều kiện a + b + c = 3 thì ta sẽ tiến hành đổi biến sang x = a − 1, y = b − 1, z = c − 1, còn nếu bất đẳng thức. số vấn đề về bất đẳng thức bậc bốn ba biến Cauchy Schwarz Để Chứng Minh Bất Đẳng Thức hai tác giả Võ Quốc Bá Cẩn, Trần Quốc Anh 6 đã đưa ra một chứng minh rất độc đáo bằng bất đẳng thức Cauchy-Schwarz như